Đến nội dung

Gachdptrai12 nội dung

Có 274 mục bởi Gachdptrai12 (Tìm giới hạn từ 08-06-2020)



Sắp theo                Sắp xếp  

#635320 VMF's Marathon Bất Đẳng Thức Olympic

Đã gửi bởi Gachdptrai12 on 24-05-2016 - 22:52 trong Bất đẳng thức và cực trị

Cách 2 cho bài 2:

Đặt $x=\sqrt{a+b}$ và $y,z$ tương tự, từ đó ta có $a=\frac{x^{2}+z^{2}-y^{2}}{2}$ và cũng tương tự cho $b,c$

Từ đó ta có

$\left | P \right |=\left | \sum \frac{a-b}{\sqrt{a+b}} \right |$

$=\left | \sum \frac{(x^{2}+z^{2}-y^{2})-(x^{2}+y^{2}-z^{2})}{2x} \right |$

$=\left |\sum \frac{(z^{2}-x^{2})}{x} \right |=\left | \sum \frac{(x-y)(y-z)(z-x)(x+y+z)}{xyz} \right |$

$=\left | \frac{(x^{2}-y^{2})(y^{2}-z^{2})(z^{2}-x^{2})}{xyz(x+y)(y+z)(z+x)} \right |$

$=\left | \frac{(a-b)(b-c)(c-a)(\sqrt{a+b}+\sqrt{b+c}+\sqrt{c+a})}{\sqrt{(a+b)(b+c)(c+a)}(\sqrt{a+b}+\sqrt{a+c})(\sqrt{b+c}+\sqrt{c+a})(\sqrt{b+c}+\sqrt{b+a})} \right |$

Đặt $c= \min \{a,b,c\}$, ta có các nhận xét 

 - $\left | (a-b)(b-c)(c-a) \right |\leq \left | ab(a+b) \right |$

 - $\frac{1}{\sqrt{(a+b)(b+c)(c+a)}}\leq \frac{1}{ab(a+b)}$

 - $\frac{1}{(\sqrt{a+b}+\sqrt{a+c})(\sqrt{b+c}+\sqrt{b+a})}\leq \frac{1}{(\sqrt{a}+\sqrt{a+b})(\sqrt{b}+\sqrt{a+b})}$

 - $\frac{\sqrt{a+b}+\sqrt{b+c}+\sqrt{c+a}}{\sqrt{b+c}+\sqrt{c+a}}= 1+\frac{\sqrt{a+b}}{\sqrt{a+c}+\sqrt{b+c}}\leq 1+ \frac{\sqrt{a+b}}{\sqrt{a}+\sqrt{b}}$

Từ các bất đẳng thức trên ta có

$\left | P \right |\leq \frac{\left | (ab(a-b)) \right |(\sqrt{a}+\sqrt{b}+\sqrt{a+b})}{\sqrt{ab(a+b)}(\sqrt{a}+\sqrt{a+b})(\sqrt{b}+\sqrt{a+b})}$

$=\left | \frac{a-b}{\sqrt{a+b}}+\sqrt{b}-\sqrt{a} \right |=\frac{1}{\sqrt{a+b+c}}\left | \frac{a-b}{\sqrt{a+b}}+\sqrt{b}-\sqrt{a} \right |$

$\leq \frac{1}{\sqrt{a+b}}\left | \frac{a-b}{\sqrt{a+b}}+\sqrt{b}-\sqrt{a} \right |=\left | \frac{a-b}{a+b}+\frac{\sqrt{b}-\sqrt{a}}{\sqrt{a+b}} \right |$

Đặt $x= \frac{a}{a+b}\Rightarrow \frac{b}{b+a}=1-x$

Suy ra $\frac{a-b}{a+b}+\frac{\sqrt{a}-\sqrt{b}}{\sqrt{a+b}}=x-(1-x)+\sqrt{1-x}-\sqrt{x}=2x-\sqrt{x}+\sqrt{1-x}-1=f(x)$

           $\Rightarrow f'(x)=2-\frac{1}{2\sqrt{x}}-\frac{1}{2\sqrt{1-x}}$

Giải phương trình $f'(x)=0$ ta có 2 nghiệm $x_{1}$ và $x_{2}$  là $\frac{8\pm \sqrt{46-2\sqrt{17}}}{16}$ và ta có $f(x_{1})\geq f(x)\geq f(x_{2})$

 Từ đó tìm được $\min$ và $\max$ của $P$




#635420 VMF's Marathon Bất Đẳng Thức Olympic

Đã gửi bởi Gachdptrai12 on 25-05-2016 - 13:34 trong Bất đẳng thức và cực trị

 

 

 

 

 Bài toán 5. (Phạm Kim Hùng) Cho các số thực $a,b,c$ dương. Tìm hằng số $k$ tốt nhất sao cho bất đẳng thức sau luôn đúng

\[\dfrac{a^3+b^3+c^3}{(a+b)(b+c)(c+a)}+\dfrac{k(ab+bc+ca)}{(a+b+c)^2}\geq \dfrac{3}{8}+\dfrac{k}{3}\]

 

Lời giải khác :

Cho $b=a=1+\sqrt{3},c=1$ ta có $k\leq \frac{9(3+2\sqrt{3})}{8}$

Ta chứng minh $k_{0}=\frac{9(3+2\sqrt{3})}{8}$ là giá trị cần tìm

Thật vậy ta sẽ chứng minh

$\dfrac{a^3+b^3+c^3}{(a+b)(b+c)(c+a)}+\dfrac{k_{0}(ab+bc+ca)}{(a+b+c)^2}\geq \dfrac{3}{8}+\dfrac{k_{0}}{3}$

Vì bđt của chúng ta thuần nhất nên ta chuẩn hóa$ a+b+c=1$ và đặt $r=abc$ và $ab+bc+ca=\frac{1-q^{2}}{3}$

Ta có bđt trở thành $\frac{3(3r+q^{2})}{-3r+1-q^{2}}+\frac{k_{0}(1-q^{2})}{3}\geq \frac{3}{8}+\frac{k_{0}}{3}$

Và đây là hàm đồng biến theo r nên nếu $2q\geq 1$ thì

$VT\geq \frac{3q^{2}}{1-q^{2}}+\frac{k_{0}(1-q^{2})}{3}\geq 1+\frac{k_{0}}{4}> \frac{3}{8}+\frac{k_{0}}{3}$

(Vì là hàm đồng biến theo $q^{2}\geq 1/4$)

Nếu $2q \leq 1$ thì ta áp dụng bổ đề của VQBC $\frac{(1+q)^{2}(1-2q)}{27}\leq r\leq \frac{(1-2q)^{2}(1+2q)}{27}$

Ta chứng minh $\frac{3((1+q)^{2}(1-2q)+9q^{2})}{-(1+q)^{2}(1-2q)+9(1-q^{2})}+\frac{k_{0}(1-q^{2})}{3}\geq \frac{3}{8}+\frac{k_{0}}{3}$

Từ đây ta phân tích được $VT-VP=\frac{3q^{2}(3+2\sqrt{3})(2\sqrt{3}-1-q)(q-2+\sqrt{3})}{8(q+1)(q-2)^{2}}\geq 0$

Hiển nhiên đúng, vậy $k_{max}=\frac{9(3+2\sqrt{3})}{8}$

p/s anh Huyện nhanh tay quá  :ohmy:  :ohmy:

 

\begin{array}{| l | l |} \hline \text{HDTterence2k} & 1\\ \hline \text{hoanglong2k} & 2\\ \hline \text{Gachdptrai12} & 2\\ \hline \text{Nguyenhuyen_AG} & 3\\ \hline \text{fatcat12345} & 1\\ \hline \text{lenhatsinh3} & 1\\ \hline\end{array} 




#635444 VMF's Marathon Bất Đẳng Thức Olympic

Đã gửi bởi Gachdptrai12 on 25-05-2016 - 15:23 trong Bất đẳng thức và cực trị

Bài toán đề nghị
Bài toán 6. Cho $a,b,c$ là các số thực thỏa mãn $ab+bc+ca=3$. Chứng minh
\[4(a^4+b^4+c^4)+11abc(a+b+c)\ge 45.\]

Viết lại bất đẳng thức về dạng thuần nhất
$4(a^4+b^4+c^4)+11abc(a+b+c)\geq 5(ab+bc+ca)^2$
$\Leftrightarrow 4(a^4+b^4+c^4)+abc(a+b+c)\geq 5(a^2b^2+b^2c^2+c^2a^2)$ (1)
Vì bđt này thuần nhất nên ta có thể bỏ điều kiện $ab+bc+ca=3$ và ta có thể chuẩn hóa $a+b+c=3$
Đặt $x+1=a,y+1=b,z+1=c$ từ đó ta có $x+y+z=0$
Thay $z=-x-y$ vào (1) ta có
$4(x^{4}+y^{4}+z^{4}+4(x^{3}+y^{3}+z^{3})+6(x^{2}+y^{2}+z^{2})+3)+ 33(xyz+xy+yz+zx+1)\geq 5(xy+yz+zx+3)^{2}$
$\Leftrightarrow 4[2(x^{2}+xy+y^{2})^{2}-12xy(x+y)+12(x^{2}+xy+y^{2})+3]+33[-xy(x+y)-(x^{2}+xy+y^{3})+1]\geq 5[3-(x^{2}+xy+y^{2})]^{2}$
$\Leftrightarrow 3(x^{2}+xy+y^{2})^{2}+45(x^{2}+xy+y^{2})\geq 81xy(x+y)$
Ta có các nhận xét sau $(x^{2}+xy+y^{2})\geq \frac{3}{4}(x+y)^{2}\geq 3xy$ nhưng bất dẳng thức đúng nếu $xy\geq0$ mà ta có $xy.xy.zx=(xyz)^2\geq0$ nên tồn tại 1 trong 3 số có 1 số lớn hơn 0, giả sử đó là xy (thỏa điểu kiện của nhận xét)
Nên ta có
$3(x^{2}+xy+y^{2})^{2}+45(x^{2}+xy+y^{2})\geq 27x^{2}y^{2}+\frac{135}{4}(x+y)^{2}$
Bây giờ ta cm
$3(x^{2}+xy+y^{2})^{2}+45(x^{2}+xy+y^{2})\geq 27x^{2}y^{2}+\frac{135}{4}(x+y)^{2}\geq81xy(x+y)$ bất đẳng thức này dễ chứng minh



#640322 UEFA EURO 2016

Đã gửi bởi Gachdptrai12 on 14-06-2016 - 19:30 trong Góc giao lưu

ANH cân hết nha :3




#645170 Tư tưởng chia để trị trong chứng minh BĐT

Đã gửi bởi Gachdptrai12 on 16-07-2016 - 16:28 trong Tài liệu, chuyên đề, phương pháp về Bất đẳng thức

ai có file ko ạ hình như file bị lỗi hết rồi tải về ko dc nữa em cảm ơn




#661962 Tìm số cách phát thỏa yêu cầu

Đã gửi bởi Gachdptrai12 on 14-11-2016 - 22:52 trong Tổ hợp - Xác suất và thống kê - Số phức

Nhà Trường có 15 cuốn sách 4 toán 5 lý 6 hóa và mỗi quyển khác nhau đôi một. Nhà trường dự định phát cho 7 học sinh số sách trên sao cho sau khi phát thì mỗi môn còn ít nhất 1 quyển . Tìm số cách phát thỏa yêu cầu 

P/s bài này có công thức tổng quát ko vậy mọi người có thì cho hỏi như thế nào và cách chứng minh 




#662673 Tìm số cách phát thỏa yêu cầu

Đã gửi bởi Gachdptrai12 on 21-11-2016 - 22:25 trong Tổ hợp - Xác suất và thống kê - Số phức

Đề bài không nói rõ là phát cho $7$ học sinh, mỗi học sinh bao nhiêu quyển (mỗi em $1$ quyển hay mỗi em từ $1$ đến $2$ quyển ?).Mình giải cho trường hợp MỖI EM $1$ QUYỂN (lần sau bạn nên đăng đề bài rõ ràng hơn)

---------------------------------------------------

Xét bài toán tổng quát :

Nhà trường có $a$ sách Toán, $b$ sách Lý, $c$ sách Hóa (các sách khác nhau từng đôi một).Nhà trường dự định phát cho $m$ học sinh ($m< a+b,m< b+c,m< a+c$), mỗi học sinh $1$ quyển sao cho mỗi môn còn lại ít nhất $1$ quyển.Hỏi có bao nhiêu cách phát thỏa yêu cầu ?

GIẢI :

Số cách chọn $m$ quyển sách sao cho không còn lại quyển Toán nào : $C_{b+c}^{m-a}$

Số cách chọn $m$ quyển sách sao cho không còn lại quyển Lý nào : $C_{a+c}^{m-b}$

Số cách chọn $m$ quyển sách sao cho không còn lại quyển Hóa nào : $C_{a+b}^{m-c}$

$\Rightarrow$ Số cách chọn $m$ quyển sao cho mỗi môn còn lại ít nhất $1$ quyển là : 

$C_{a+b+c}^m-\left ( C_{b+c}^{m-a}+C_{a+c}^{m-b}+C_{a+b}^{m-c} \right )$

Số cách phát thỏa yêu cầu là $\left [ C_{a+b+c}^m-\left ( C_{b+c}^{m-a}+C_{a+c}^{m-b}+C_{a+b}^{m-c} \right ) \right ].m!$

Lưu ý quy ước : $C_p^q=0$ nếu $q< 0$

 

Thay số vào, số cách phát là $\left [ C_{15}^7-(C_{11}^3+C_{10}^2+C_9^1) \right ].7!=31328640$ (cách).

cho em hỏi là mấy cái tổng quát này là hay có ở đâu vậy anh em cảm ơn 




#593321 Toppic Các bài toán BĐT qua các kì thi olympic 30/4

Đã gửi bởi Gachdptrai12 on 11-10-2015 - 21:17 trong Bất đẳng thức - Cực trị

Đặt $$C=\dfrac{a}{\sqrt{7a^2+b^2+c^2}}+\dfrac{b}{\sqrt{a^2+7b^2+c^2}}+\dfrac{c}{\sqrt{a^2+b^2+7c^2}}$$
và $$D=a(7a^2+b^2+c^2)+b(a^2+7b^2+c^2)+c(a^2+b^2+7c^2)$$
Áp dụng bất đẳng thức $holder$ , ta có :
$$C^2.D \geq (a+b+c)^3$$
Nên ta cần chứng minh $(a+b+c)^3 \geq D$
Ta có : $$D=7(a+b+c)+(a+b+c)(ab+bc+ca)-3 \geq 7(a+b+c)+\dfrac{(a+b+c)^3}{3}-3 \geq (a+b+c)^3$$




#652188 Topic yêu cầu tài liệu THCS

Đã gửi bởi Gachdptrai12 on 01-09-2016 - 00:12 trong Tài liệu - Đề thi

cho em xin cái tài liệu CYH của anh Cẩn =)))




#661986 Topic yêu cầu tài liệu Olympic

Đã gửi bởi Gachdptrai12 on 15-11-2016 - 10:23 trong Tài nguyên Olympic toán

mọi người có tài liệu về  "điểm bất động" với "mất thứ tự" trong tổ hợp không vậy cho mình xin với cảm ơn nhiều!!




#608560 Topic về bất đẳng thức

Đã gửi bởi Gachdptrai12 on 11-01-2016 - 22:22 trong Bất đẳng thức - Cực trị

Dùng bất đẳng thức này để giải sẽ nhanh hơn
 
\[27(a^2b+b^2c+c^2a+abc) \leqslant 4(a+b+c)^3,\]
với $a,\,b,\,c$ không âm.

He he bổ đề anh huyện nói cũng khó nhai đó nhân ra hết hoặc dùng phép E.M.V của anh P.K.H bài này bên LQD đà nẵng đã ra 1 lần bài này
P/s Mà anh huyện giải ra bài em đưa trên face chưa z



#663230 Topic về bất đẳng thức

Đã gửi bởi Gachdptrai12 on 27-11-2016 - 20:56 trong Bất đẳng thức - Cực trị

Cho a,b,c là các số thực dương thỏa mãn: abc=1, chứng minh rằng:

$ab^2+bc^2+ca^2\ge ab+bc+ca$

Đổi biến $(a,b,c)->(\frac{a}{b},\frac{b}{c},\frac{c}{a})$ Bất đẳng thức trở thành

$\sum \frac{a^{2}}{bc}\geq \sum \frac{a}{c}\Leftrightarrow \sum a^{3}\geq \sum a^{2}b$ hiển nhiên là AM-GM




#627210 Topic tập hợp các BĐT đánh giá từng biến

Đã gửi bởi Gachdptrai12 on 15-04-2016 - 09:26 trong Bất đẳng thức - Cực trị

 

Đề bài 6

 

Cho x, y và z là ba số thực dương, thoả mãn:

 

$x^{2}+y^{2}+z^{2}=2\left ( xy+yz+zx \right ).$
 
Chứng minh rằng:
 
$\frac{x}{y}+\frac{y}{z}+\frac{z}{x}\geq 5.$

 

bài ni có 1 ý tưởng đó là dùng bổ đề hoán vị nhưng thấy rắc rối quá và cái bổ đề cũng khó chứng minh nữa :3 @@ ta cũng có thể tổng quát lên thành đề VMEO tháng 11 THPT :)))

p/s ai có thể dùng phân tích p,q,r giải dc ko vại 

một bài toán thú vị khác

 

Bài 7:  Cho n số thực$ x_1,x_2,...,x_n thoả mãn x_1+x_2+...+x_n=n$. Đặt :

 
$x_1^2+x_2^2+...+x_n^2=n+n(n-1)t^2$
 
với $t\geq 0$. Khi đó ta có :
 
$1-(n-1)t\leq x_i\leq 1+(n-1)t,\;\forall i=\overline{1,n}$



#614821 Tiếp sức bất đẳng thức

Đã gửi bởi Gachdptrai12 on 14-02-2016 - 08:08 trong Bất đẳng thức và cực trị

cho hỏi bất đẳng thức này có thể chuẩn hóa được không ạ

được bạn vì nó thuần nhất bậc 1 :v @@




#616805 Tiếp sức bất đẳng thức

Đã gửi bởi Gachdptrai12 on 25-02-2016 - 09:44 trong Bất đẳng thức và cực trị

Bài 86:cho $x,y$ là các số thực thỏa $x+y+2=2(\sqrt{x-1} + \sqrt{y+2})$. Tìm min $\frac{1}{x}+\frac{1}

baì này mình làm thế này ko biết đúng hay không thấy đăng mà không ai giải 

áp dụng bđt C-S ta có$2(\sqrt{x-1}+\sqrt{y+2})\geq 2\sqrt{2(x+y+1)}$

đặt x+y+1=t ta có từ giả thuyêt=>t+1$\geq 2\sqrt{2(t)}$ ta chăn được t rồi ta có 

$\frac{1}{x}+\frac{1}{y+3}\geq \frac{4}{x+y+3}=\frac{1}{t+2}$ => ra được min mà vừa đảm bảo được dấu bằng




#615938 Tiếp sức bất đẳng thức

Đã gửi bởi Gachdptrai12 on 19-02-2016 - 20:45 trong Bất đẳng thức và cực trị

Ta chỉ cần chứng minh
\[(a+b+c)^3 \geqslant 6\sqrt{3}|(a-b)(b-c)(c-a)|.\]
Giả sử $a \geqslant b \geqslant c.$ Áp dụng bất đẳng thức AM-GM, ta có
\[\begin{aligned} \left |(a-b)(b-c)(c-a)\right| &=(a-c)(b-c)(a-b)\\&\le (a+c)\cdot b \cdot (a+c-b)\\&=\frac{1}{2} \cdot \left (1+\sqrt{3} \right )(a+c)\cdot b \left ( -1+\sqrt{3} \right ) \cdot (a+c-b)\\& \le \frac{1}{2}\left [ \frac{\left (1+\sqrt{3} \right )(a+c)+ b \left ( -1+\sqrt{3} \right ) + (a+c-b)}{3} \right ]^3\\& =\frac{1}{6\sqrt{3}}(a+b+c)^3.\end{aligned}\]
Từ đó suy ra điều phải chứng minh.

bài này anh huyện làm thế nào ấy thấy lẫn cái căn zô khó khó nghĩ tới :) sao ấy tối em đăng lời giải của em mong anh cho nhận xét(srry tối ni tự nhiên chị không đem máy tính zề đói meo rồi hồi sửa mệt bà chị) :(



#613879 Tiếp sức bất đẳng thức

Đã gửi bởi Gachdptrai12 on 10-02-2016 - 09:43 trong Bất đẳng thức và cực trị

Anh có viết một chuyên đề trong đó có nhắc đến bài toán này, em xem trong file đính kèm nhé. :)

:)

dạ em có files rồi em xem trên juliel TV ấy blog hay quá trời lun à :v @@ :v

một bài khác cũng khá hay 

Bài 60:cho a,b,c là các số thực dương thỏa x2+y2+z2=1 chứng minh 

$1\leq \frac{x}{1+yz}+\frac{y}{1+xz}+\frac{z}{1+xy}\leq \sqrt{2}$




#615836 Tiếp sức bất đẳng thức

Đã gửi bởi Gachdptrai12 on 19-02-2016 - 10:38 trong Bất đẳng thức và cực trị

Bài 82: Cho $a,b,c \geq 0$ thỏa mãn $a+b+c=3$. Chứng minh
$(a-b)(b-c)(c-a)\leq \frac{3\sqrt{3}}{2}$
 




#615054 Tiếp sức bất đẳng thức

Đã gửi bởi Gachdptrai12 on 14-02-2016 - 21:04 trong Bất đẳng thức và cực trị

Bài 77: cho a,b,c dương cm$a^{3}+b^{3}+c^{3}+3abc\frac{a^{2}b+b^{2}c+c^{2}a}{ab^{2}+bc^{2}+ca^{2}}\geq ab(a+b)+bc(b+c)+ca(c+a)$




#615190 Tiếp sức bất đẳng thức

Đã gửi bởi Gachdptrai12 on 15-02-2016 - 17:48 trong Bất đẳng thức và cực trị

Đề này có phần màu đỏ không bạn ?Nếu không có là Schur bậc 3 rồi :D
Còn nếu có thì chỉ cần giả sử $c\geq a\geq b$ thì $VT\geq a^{3}+b^{3}+c^{3}+3abc\geq ab(a+b)+bc(b+c)+ca(c+a)$ cũng là Schur bậc 3 :D

Bài 77: Áp dụng thẳng BĐT Schur bậc 3 : $a^{3}+b^{3}+c^{3}+3abc\geq ab(a+b)+bc(b+c)+ca(c+a)$ Chỉ cần chứng minh được $a^{2}b+b^{2}c+c^{2}a\geq ab^{2}+bc^{2}+ca^{2}$ là bài toán hoàn tất! Thật vậy, thực ra BĐT trên là khai triển khá quen thuộc:  Không mất tính tổng quát: giả sử $a\geq b\geq c$ ta có: $(a-b)(b-c)(a-c)\geq 0\Leftrightarrow a^{2}b+b^{2}c+c^{2}a\geq ab^{2}+bc^{2}+ca^{2}$ .............................................

file đây và PP S.O.C nếu các bạn giả sử thế thì phải làm 2TH luôn bđt này ko đối xứng nên bạn chỉ được g/s 2 TH

File gửi kèm




#614129 Tiếp sức bất đẳng thức

Đã gửi bởi Gachdptrai12 on 11-02-2016 - 11:12 trong Bất đẳng thức và cực trị

 

:)

dạ em có files rồi em xem trên juliel TV ấy blog hay quá trời lun à :v @@ :v

một bài khác cũng khá hay 

Bài 60:cho a,b,c là các số thực dương thỏa x2+y2+z2=1 chứng minh 

 

bài này làm thế này 

VT) $x+yz\leq x+\frac{x(y^{2}+z^{2})}{2}=x+\frac{x(1-x^{2})}{2}=-(x+2)(x-1)^{2}+1\leq 1$ta sẽ cm bổ đề sau 

$1\leq \frac{x}{1+yz}+\frac{y}{1+xz}+\frac{z}{1+xy}\leq \sqrt{2}$

=>x+xyz<=1 tương tự y+xyz<=1,z+xyz<=1 
do đó $\sum \frac{x^{2}}{x+xyz}\geq x^{2}+y^{2}+z^{2}=1$
VP) ta có 1 bổ đề tiếp ta chứng minh 
$1+yz\geq \frac{x+y+z}{\sqrt{2}}\Leftrightarrow 2(1+yz)^{2}\geq (x+y+z)^{2}\Leftrightarrow (x-y-z)^{2}+2y^{2}z^{2}\geq 0$
tương tự cho xy+1,xz+1 rồi ta có cộng các bđt trên lại thì có dpcm :v



#615178 Tiếp sức bất đẳng thức

Đã gửi bởi Gachdptrai12 on 15-02-2016 - 17:13 trong Bất đẳng thức và cực trị

Đề này có phần màu đỏ không bạn ?Nếu không có là Schur bậc 3 rồi :D
Còn nếu có thì chỉ cần giả sử $c\geq a\geq b$ thì $VT\geq a^{3}+b^{3}+c^{3}+3abc\geq ab(a+b)+bc(b+c)+ca(c+a)$ cũng là Schur bậc 3 :D

bạn làm rứa thiếu TH a>=b>=c đó bđt này nói ra cũng thú vị dùng S.O.C hoặc cách khác nhưng S.O.C là Hay nhất



#613845 Tiếp sức bất đẳng thức

Đã gửi bởi Gachdptrai12 on 09-02-2016 - 23:20 trong Bất đẳng thức và cực trị

một bài mình tâm đắc nè 

Bài 56: cho x,y,z là các số thực c/m

$((x-y)^{3}+(y-z)^{3}+(z-x)^{3})(\frac{1}{(x-y)^{3}}+\frac{1}{(y-z)^{3}}+\frac{1}{(z-x)^{3}})\leq \frac{-45}{4}$




#616267 Tiếp sức bất đẳng thức

Đã gửi bởi Gachdptrai12 on 21-02-2016 - 18:02 trong Bất đẳng thức và cực trị

Bài 86:cho $x,y$ là các số thực thỏa $x+y+2=2(\sqrt{x-1} + \sqrt{y+2})$. Tìm min $\frac{1}{x}+\frac{1}{y+3}$




#616377 Tiếp sức bất đẳng thức

Đã gửi bởi Gachdptrai12 on 22-02-2016 - 10:43 trong Bất đẳng thức và cực trị

Ta chỉ cần chứng minh

\[(a+b+c)^3 \geqslant 6\sqrt{3}|(a-b)(b-c)(c-a)|.\]

Giả sử $a \geqslant b \geqslant c.$ Áp dụng bất đẳng thức AM-GM, ta có
\[\begin{aligned} \left |(a-b)(b-c)(c-a)\right| &=(a-c)(b-c)(a-b)\\&\le (a+c)\cdot b \cdot (a+c-b)\\&=\frac{1}{2} \cdot \left (1+\sqrt{3} \right )(a+c)\cdot b \left ( -1+\sqrt{3} \right ) \cdot (a+c-b)\\& \le \frac{1}{2}\left [ \frac{\left (1+\sqrt{3} \right )(a+c)+ b \left ( -1+\sqrt{3} \right ) + (a+c-b)}{3} \right ]^3\\& =\frac{1}{6\sqrt{3}}(a+b+c)^3.\end{aligned}\]
Từ đó suy ra điều phải chứng minh.

bình phương => ta chứng minh bđt manh hơn (a-b)2(b-c)2(c-a)2$\leq \frac{27}{4}$ ta có$(b-c)^{2}\leq b^{2},(a-c)^{2}\leq a^{2}$

ta chứng minh$4a^{2}b^{2}(a-b)^{2}\leq 27$ áp dụng AM-GM ta có $4a^{2}b^{2}(a-b)^{2}\leq (\frac{2ab+2ab+(a-b)^{3}}{27})$ =$\frac{(a+b)^{6}}{27}$ mà a+b<=a+b+c=3 +. dpcm